You are on page 1of 20

Chapter IV: THE SEARCH AND SEIZURE PROVISION

SEC. 18. Period of Detention Without Judicial Warrant of Arrest. The


provisions of Article 125 of the Revised Penal Code to the contrary
notwithstanding, any police or law enforcement personnel, who, having been
duly authorized in writing by the Anti-Terrorism Council has taken custody
of a person charged with or suspected of the crime of terrorism or the crime
of conspiracy to commit terrorism shall, without incurring any criminal
liability for delay in the delivery of detained persons to the proper judicial
authorities, deliver said charged or suspected person to the proper judicial
authority within a period of three (3) days counted from the moment the said
charged or suspected person has been apprehended or arrested, detained,
and taken into custody by the said police, or law enforcement personnel:
Provided, That the arrest of those suspected of the crime of terrorism or
conspiracy to commit terrorism must result from the surveillance under
Section 7 and examination of bank deposits under Section 27 of this Act.
The police or law enforcement personnel concerned shall, before detaining
the person suspected of the crime of terrorism, present him or her before any
judge at the latters residence or office nearest the place where the arrest
took place at any time of the day or night. It shall be the duty of the judge,
among other things, to ascertain the identity of the police or law enforcement
personnel and the person or persons they have arrested and presented before
him or her, to inquire of them the reasons why they have arrested the person
and determine by questioning and personal observation whether or not the
suspect has been subjected to any physical, moral or psychological torture by
whom and why. The judge shall then submit a written report of what he/she
had observed when the subject was brought before him to the proper court
that has jurisdiction over the case of the person thus arrested. the judge
shall forthwith submit his/her report within three (3) calendar days from the
time the suspect was brought to his/her residence or office.
Immediately after taking custody of a person charged with or suspected of
the crime of terrorism or conspiracy to commit terrorism, the police or law
enforcement personnel shall notify in writing the judge of the court nearest
the place of apprehension or arrest: Provided, That where the arrest is made
during saturdays, sundays, holidays or after office hours, the written notice
shall be served at the residence of the judge nearest the place where the
accused was arrested.
The penalty of ten (10) years and one day to twelve (12) years of
imprisonment shall be imposed upon the police or law enforcement personnel
who fails to notify any judge as provided in the preceding paragraph.
SEC. 19. Period of Detention in the Event of an Actual or Imminent Terrorist
Attack. In the event of an actual or imminent terrorist attack, suspects
may not be detained for more than three (3) days without the written
approval of a municipal, city, provincial or regional official of a Human Rights

Commission or judge of the municipal, regional trial court, the


Sandiganbayan or a justice of the Court of Appeals nearest the place of the
arrest. If the arrest is made during Saturdays, Sundays, holidays or after
office hours, the arresting police or law enforcement personnel shall bring
the person thus arrested to the residence of any of the officials mentioned
above that is nearest the place where the accused was arrested. The approval
in writing of any of the said officials shall be secured by the police or law
enforcement personnel concerned within five (5) days after the date of the
detention of the persons concerned: Provided, however, That within three (3)
days after the detention the suspects, whose connection with the terror
attack or threat is not established, shall be released immediately.
SEC. 26. Restriction on Travel. In cases where evidence of guilt is not
strong, and the person charged with the crime of terrorism or conspiracy to
commit terrorism is entitled to bail and is granted the same, the court, upon
application by the prosecutor, shall limit the right of travel of the accused to
within the municipality or city where he resides or where the case is pending,
in the interest of national security and public safety, consistent with Article
III, Section 6 of the Constitution. Travel outside of said municipality or city,
without the authorization of the court, shall be deemed a violation of the
terms and conditions of his bail, which shall then be forfeited as provided
under the Rules of Court.
He or she may also be placed under house arrest by order of the court at his
or her usual place of residence.
While under house arrest, he or she may not use telephones, cellphones, emails, computers, the internet or other means of communications with
people outside the residence until otherwise ordered by the court.
The restrictions abovementioned shall be terminated upon the acquittal of
the accused or of the dismissal of the case filed against him or earlier upon
the discretion of the court on motion of the prosecutor or of the accused.
REQUISITES OF A VALID SEARCH WARRANT
Frank Uy & Unifish Packing Corp. vs Bureau of Internal Revenue et al
Search and Seizure Requisites of a Valid Search Warrant
In Sept 1993, Rodrigo Abos, a former employee of UPC reported to the BIR
that Uy Chin Ho aka Frank Uy, manager of UPC, was selling thousands of
cartons of canned cartons without issuing a report. This is a violation of Sec
253 & 263 of the Internal Revenue Code. In Oct 1993, the BIR requested
before RTC Cebu to issue a search warrant. Judge Gozo-Dadole issued a
warrant on the same day. A second warrant was issued which contains the
same substance but has only one page, the same was dated Oct 1 st 2003.
These warrants were issued for the alleged violation by Uy of Sec 253. A third
warrant was issued on the same day for the alleged violation of Uy of Sec 238
in relation to sec 263. On the strength of these warrants, agents of the BIR,
accompanied by members of the PNP, on 2 Oct 1993, searched the premises

of the UPC. They seized, among other things, the records and documents of
UPC. A return of said search was duly made by Labaria with the RTC of
Cebu. UPC filed a motion to quash the warrants which was denied by the
RTC. They appealed before the CA via certiorari. The CA dismissed the
appeal for a certiorari is not the proper remedy.
ISSUE: Whether or not there was a valid search warrant issued.
HELD: The SC ruled in favor of UPC and Uy in a way for it ordered the return
of the seized items but sustained the validity of the warrant. The SC ruled
that the search warrant issued has not met some basic requisites of validity.
A search warrant must conform strictly to the requirements of the foregoing
constitutional and statutory provisions. These requirements, in outline form,
are:
(1) the warrant must be issued upon probable cause;
(2) the probable cause must be determined by the judge himself and not by
the applicant or any other person;
(3) in the determination of probable cause, the judge must examine, under
oath or affirmation, the complainant and such witnesses as the latter may
produce; and
(4) the warrant issued must particularly describe the place to be searched
and persons or things to be seized.
The SC noted that there has been inconsistencies in the description of the
place to be searched as indicated in the said warrants. Also the thing to be
seized was not clearly defined by the judge. He used generic itineraries. The
warrants were also inconsistent as to who should be searched. One warrant
was directed only against Uy and the other was against Uy and UPC. The SC
however noted that the inconsistencies wered cured by the issuance of the
latter warrant as it has revoked the two others.
Section 2, Article III of the Constitution guarantees the right of the people
against unreasonable searches and seizures:
The right of the people to be secure in their persons, houses, papers, and
effects against unreasonable searches and seizures of whatever nature and
for any purpose shall be inviolable, and no search warrant or warrant of
arrest shall issue except upon probable cause to be determined personally by
the judge after examination under oath or affirmation of the complainant
and the witnesses he may produce, and particularly describing the place to
be searched and the persons or things to be seized.
NOTES
Rule 126 of the Rules of Court provides:
SEC. 3. Requisite for issuing search warrant. A search warrant shall not
issue but upon probable cause in connection with one specific offense to be
determined personally by the judge after examination under oath or
affirmation of the complainant and the witnesses he may produce, and
particularly describing the place to be searched and the things to be seized.

SEC. 4. Examination of complainant; record. The judge must, before


issuing the warrant, personally examine in the form of searching questions
and answers, in writing and under oath the complainant and any witnesses
he may produce on facts personally known to them and attach to the record
their sworn statements together with any affidavits submitted.
a. The place to be searched in the warrant is controlling
People vs Court of Appeals (291 SCRA 400)
FACTS: A petition for certiorari has been filed to invalidate the order of Judge
Casanova which quashed search warrant issued by Judge Bacalla and
declared inadmissible for any purpose the items seized under the warrant.
>An application for a search warrant was made by S/Insp Brillantes against
Mr. Azfar Hussain who had allegedly in his possession firearms and
explosives at Abigail Variety Store, Apt 1207 Area F. Bagon Buhay Avenue,
Sarang Palay, San Jose Del Monte, Bulacan. The following day Search
Warrant No. 1068 was issued but was served not at Abigail Variety Store but
at Apt. No. 1, immediately adjacent to Abigail Variety Store resulting in the
arrest of 4 Pakistani nationals and the seizure of a number of different
explosives and firearms.
ISSUE: WON a search warrant was validly issued as regard the apartment in
which private respondents were then actually residing, or more explicitly,
WON that particular apartment had been specifically described in the
warrant.
HELD: The ambiguity lies outside the instrument, arising from the absence
of a meeting of minds as to the place to be searched between the applicants
for the warrant and the Judge issuing the same; and what was done was to
substitute for the place that the Judge had written down in the warrant, the
premises that the executing officers had in their mind. This should not have
been done. It is neither fair nor licit to allow police officers to search a place
different from that stated in the warrant on the claim that the place actually
searched although not that specified in the warrant is exactly what they
had in view when they applied for the warrant and had demarcated in their
supporting evidence. What is material in determining the validity of a search
is the place stated in the warrant itself, not what the applicants had in their
thoughts, or had represented in the proofs they submitted to the court
issuing the warrant.
The place to be searched, as set out in the warrant, cannot be amplified or
modified by the officers'
own personal knowledge of the premises, or the evidence they adduced in
support of their application for the warrant. Such a change is proscribed by
the Constitution which requires inter alia the search warrant to particularly
describe the place to be searched as well as the persons or things to be
seized. It would concede to police officers the power of choosing the place to
be searched, even if it not be that delineated in the warrant. It would open

wide the door to abuse of the search process, and grant to officers executing
a search warrant that discretion which the Constitution has precisely
removed from them. The particularization of the description of the place to be
searched may properly be done only by the Judge, and only in the warrant
itself; it cannot be left to the discretion of the police officers conducting the
search.
In applying for a search warrant, the police officers had in their mind
the first four (4) separate apartment units at the rear of ABIGAIL
VARIETY STORE in Quezon City to be the subject of their search. The
same was not, however, what the Judge who issued the warrant had in
mind, AND WAS NOT WHAT WAS ULTIMATELY DESCRIBED IN THE
SEARCH WARRANT. As such, any evidence obtained from the place
searched which is different from that indicated in the search warrant is
inadmissible in evidence for any purpose and in any proceeding.
This is so because it is neither licit nor fair to allow police officers to
search a place different from that stated in the warrant on the claim
that the place actually searchedalthough not that specified in the
search warrantis exactly what they had in view when they applied for
the warrant and had demarcated in their supporting evidence. WHAT IS
MATERIAL IN DETERMINING THE VALIDITY OF A SEARCH IS THE
PLACE STATED IN THE WARRANT ITSELF, NOT WHAT THE APPLICANTS
HAD IN THEIR THOUGHTS, OR HAD REPRESENTED IN THE PROOFS
THEY SUBMITTED TO THE COURT ISSUING THE WARRANT. As such, it
was not just a case of obvious typographical error, but a clear case of
a search of a place different from that clearly and without ambiguity
identified in the search warrant.
NOTE: Very Important: Where a search warrant is issued by one court
and the criminal action based on the results of the search is afterwards
commenced in another court, IT IS NOT THE RULE THAT A MOTION TO
QUASH THE WARRANT (or to retrieve the things seized) MAY BE FILED
ONLY IN THE ISSUING COURTSUCH A MOTION MAY BE FILED FOR
THE FIRST TIME IN EITHER THE ISSUING COURT OR THAT IN WHICH
THE CRIMINAL PROCEEDING IS PENDING.
b. Validity of a warrantless search and seizure as a result of an
informers tip
1. People of the Philippines vs Rosa Aruta y Menguin
Search and Seizure Informers Tip
In the morning of 13 Dec 1988, the law enforcement officers received
information from an informant named Benjie that a certain Aling Rosa
would be leaving for Baguio City on 14 Dec 1988 and would be back in the
afternoon of the same day carrying with her a large volume of marijuana; At

6:30 in the evening of 14 Dec 1988, Aruta alighted from a Victory Liner Bus
carrying a travelling bag even as the informant pointed her out to the law
enforcement officers; NARCOM officers approached her and introduced
themselves as NARCOM agents; When asked by Lt. Abello about the
contents of her travelling bag, she gave the same to him; When they opened
the same, they found dried marijuana leaves; Aruta was then brought to the
NARCOM office for investigation.
ISSUE: Whether or not the conducted search and seizure is constitutional.
HELD: The SC ruled in favor of Aruta and has noted that some drug
traffickers are being freed due to technicalities. Aruta cannot be said to be
committing a crime. Neither was she about to commit one nor had she just
committed a crime. Aruta was merely crossing the street and was not acting
in any manner that would engender a reasonable ground for the NARCOM
agents to suspect and conclude that she was committing a crime. It was only
when the informant pointed to Aruta and identified her to the agents as the
carrier of the marijuana that she was singled out as the suspect. The
NARCOM agents would not have apprehended Aruta were it not for the
furtive finger of the informant because, as clearly illustrated by the evidence
on record, there was no reason whatsoever for them to suspect that accusedappellant was committing a crime, except for the pointing finger of the
informant. The SC could neither sanction nor tolerate as it is a clear violation
of the constitutional guarantee against unreasonable search and seizure.
Neither was there any semblance of any compliance with the rigid
requirements of probable cause and warrantless arrests. Consequently, there
was no legal basis for the NARCOM agents to effect a warrantless search of
Arutas bag, there being no probable cause and the accused-appellant not
having been lawfully arrested. Stated otherwise, the arrest being incipiently
illegal, it logically follows that the subsequent search was similarly illegal, it
being not incidental to a lawful arrest. The constitutional guarantee against
unreasonable search and seizure must perforce operate in favor of accusedappellant. As such, the articles seized could not be used as evidence against
accused-appellant for these are fruits of a poisoned tree and, therefore,
must be rejected, pursuant to Article III, Sec. 3(2) of the Constitution.
NOTES:
When is a warrantless search allowed?
1. Warrantless search incidental to a lawful arrest recognized under Section
12, Rule 126 of the Rules of Court 8 and by prevailing jurisprudence;
2. Seizure of evidence in plain view, the elements of which are:
(a) a prior valid intrusion based on the valid warrantless arrest in which the
police are legally present in the pursuit of their official duties;
(b) the evidence was inadvertently discovered by the police who had the right
to be where they are;
(c) the evidence must be immediately apparent, and
(d) plain view justified mere seizure of evidence without further search;

3. Search of a moving vehicle. Highly regulated by the government, the


vehicles inherent mobility reduces expectation of privacy especially when its
transit in public thoroughfares furnishes a highly reasonable suspicion
amounting to probable cause that the occupant committed a criminal
activity;
4. Consented warrantless search;
5. Customs search;
6. Stop and Frisk; and
7. Exigent and Emergency Circumstances.
2. The People of the Philippines vs Ruben Montilla y Gatdula
Political Law Search and Seizure Informers Tip Warrantless Arrest
On 19 June 1994 at about 2pm, police officers Talingting and Clarin were
informed by an asset that a drug courier would be arriving from Baguio to
Dasmarias carrying an undetermined amount of marijuana. The next day,
the informant pointed at Montilla as the courier who was waiting in a waiting
shed Brgy Salitran, Dasmarias. Montilla was then apprehended and he was
caught in possession of a bag and a carton worth 28 kilos of marijuana.
Montilla denied the allegation and he said he came to Cavite from Baguio for
work and he does not have any effects with him at that time except for some
pocket money. He was sentenced to death thereafter. He averred that the
search and seizure conducted was illegal for there was no warrant and that
he should have been given the opportunity to cross examine the informant.
He said that if the informant has given the cops the information about his
arrival as early as the day before his apprehension, the cops should have
ample time to secure a search warrant.
ISSUE: Whether or not the warrantless arrest conducted is legal.
HELD: The SC ruled that the warrantless arrest is legal and so was the
warrantless search. Sec 2 Art 3 of the Constitution has its exception when it
comes to warrantless searches, they are:
(1) customs searches;
(2) searches of moving vehicles,
(3) seizure of evidence in plain view;
(4) consented searches;
(5) searches incidental to a lawful arrest;
(6) stop and frisk measures have been invariably recognized as the
traditional exceptions.
In the case at bar, it should be noted that the information relayed by
informant to the cops was that there would be delivery of marijuana at
Barangay Salitran by a courier coming from Baguio in the early morning of
June 20, 1994. Even assuming that the policemen were not pressed for time,
this would be beside the point for, under these circumstances, the
information relayed was too sketchy and not detailed enough for the
obtention of the corresponding arrest or search warrant. While there is an

indication that the informant knew the courier, the records do not reveal that
he knew him by name.
On such bare information, the police authorities could not have properly
applied for a warrant, assuming that they could readily have access to a
judge or a court that was still open by the time they could make preparations
for applying therefor, and on which there is no evidence presented by the
defense. In determining the opportunity for obtaining warrants, not only the
intervening time is controlling but all the coincident and ambient
circumstances should be considered, especially in rural areas.
A legitimate warrantless arrest, as above contemplated, necessarily cloaks
the arresting police officer with authority to validly search and seize from the
offender
(1) dangerous weapons, and
(2) those that may be used as proof of the commission of an offense.
3. People V Racho
On appeal is the Court of Appeals (CA) Decision 1 dated May 22, 2008 in CAG.R. CR-H.C. No. 00425 affirming the Regional Trial Court 2 (RTC) Joint
Decision3 dated July 8, 2004 finding appellant Jack Racho y Raquero guilty
beyond reasonable doubt of Violation of Section 5, Article II of Republic Act
(R.A.) No. 9165.
The case stemmed from the following facts:
On May 19, 2003, a confidential agent of the police transacted through
cellular phone with appellant for the purchase of shabu. The agent later
reported the transaction to the police authorities who immediately formed a
team composed of member of the Philippine Drug Enforcement Agency
(PDEA), the Intelligence group of the Philippine Army and the local police
force to apprehend the appellant. 4 The agent gave the police appellants
name, together with his physical description. He also assured them that
appellant would arrive in Baler, Aurora the following day.
On May 20, 2003, at 11:00 a.m., appellant called up the agent and informed
him that he was on board a Genesis bus and would arrive in Baler, Aurora,
anytime of the day wearing a red and white striped T-shirt. The team
members then posted themselves along the national highway in Baler,
Aurora. At around 3:00 p.m. of the same day, a Genesis bus arrived in Baler.
When appellant alighted from the bus, the confidential agent pointed to him
as the person he transacted with earlier. Having alighted from the bus,
appellant stood near the highway and waited for a tricycle that would bring
him to his final destination. As appellant was about to board a tricycle, the
team approached him and invited him to the police station on suspicion of
carrying shabu. Appellant immediately denied the accusation, but as he
pulled out his hands from his pants pocket, a white envelope slipped
therefrom which, when opened, yielded a small sachet containing the
suspected drug.5

The team then brought appellant to the police station for investigation. The
confiscated specimen was turned over to Police Inspector Rogelio Sarenas De
Vera who marked it with his initials and with appellants name. The field test
and laboratory examinations on the contents of the confiscated sachet
yielded positive results for methamphetamine hydrochloride. 6
Appellant was charged in two separate Informations, one for violation of
Section 5 of R.A. 9165, for transporting or delivering; and the second, of
Section 11 of the same law for possessing, dangerous drugs, the accusatory
portions of which read:
"That at about 3:00 oclock (sic) in the afternoon on May 20, 2003 in Baler,
Aurora and within the jurisdiction of this Honorable Court, the said accused,
did then and there, unlawfully, feloniously and willfully have in his
possession five point zero one (5.01) [or 4.54] grams of Methamphetamine
Hydrochloride commonly known as "Shabu", a regulated drug without any
permit or license from the proper authorities to possess the same.
CONTRARY TO LAW."7
"That at about 3:00 oclock (sic) in the afternoon on May 20, 2003 in Baler,
Aurora, the said accused did then and there, unlawfully, feloniously and
willfully transporting or delivering dangerous drug of 5.01 [or 4.54] grams of
shabu without any permit or license from the proper authorities to transport
the same.
CONTRARY TO LAW."8
During the arraignment, appellant pleaded "Not Guilty" to both charges.
At the trial, appellant denied liability and claimed that he went to Baler,
Aurora to visit his brother to inform him about their ailing father. He
maintained that the charges against him were false and that no shabu was
taken from him. As to the circumstances of his arrest, he explained that the
police officers, through their van, blocked the tricycle he was riding in; forced
him to alight; brought him to Sea Breeze Lodge; stripped his clothes and
underwear; then brought him to the police station for investigation. 9
On July 8, 2004, the RTC rendered a Joint Judgment 10 convicting appellant
of Violation of Section 5, Article II, R.A. 9165 and sentencing him to suffer
the penalty of life imprisonment and to pay a fine of P500,000.00; but
acquitted him of the charge of Violation of Section 11, Article II, R.A. 9165.
On appeal, the CA affirmed the RTC decision. 11
Hence, the present appeal.
In his brief,12 appellant attacks the credibility of the witnesses for the
prosecution. He likewise avers that the prosecution failed to establish the
identity of the confiscated drug because of the teams failure to mark the
specimen immediately after seizure. In his supplemental brief, appellant
assails, for the first time, the legality of his arrest and the validity of the
subsequent warrantless search. He questions the admissibility of the
confiscated sachet on the ground that it was the fruit of the poisonous tree.
The appeal is meritorious.

We have repeatedly held that the trial courts evaluation of the credibility of
witnesses and their testimonies is entitled to great respect and will not be
disturbed on appeal. However, this is not a hard and fast rule. We have
reviewed such factual findings when there is a showing that the trial judge
overlooked, misunderstood, or misapplied some fact or circumstance of
weight and substance that would have affected the case. 13
Appellant focuses his appeal on the validity of his arrest and the search and
seizure of the sachet of shabu and, consequently, the admissibility of the
sachet. It is noteworthy that although the circumstances of his arrest were
briefly discussed by the RTC, the validity of the arrest and search and the
admissibility of the evidence against appellant were not squarely raised by
the latter and thus, were not ruled upon by the trial and appellate courts.
It is well-settled that an appeal in a criminal case opens the whole case for
review.1avvphi1 This Court is clothed with ample authority to review matters,
even those not raised on appeal, if we find them necessary in arriving at a
just disposition of the case. Every circumstance in favor of the accused shall
be considered. This is in keeping with the constitutional mandate that every
accused shall be presumed innocent unless his guilt is proven beyond
reasonable doubt.14
After a thorough review of the records of the case and for reasons that will be
discussed below, we find that appellant can no longer question the validity of
his arrest, but the sachet of shabu seized from him during the warrantless
search is inadmissible in evidence against him.
The records show that appellant never objected to the irregularity of his
arrest before his arraignment. In fact, this is the first time that he raises the
issue. Considering this lapse, coupled with his active participation in the
trial of the case, we must abide with jurisprudence which dictates that
appellant, having voluntarily submitted to the jurisdiction of the trial court,
is deemed to have waived his right to question the validity of his arrest, thus
curing whatever defect may have attended his arrest. The legality of the
arrest affects only the jurisdiction of the court over his person. Appellants
warrantless arrest therefore cannot, in itself, be the basis of his acquittal. 15
As to the admissibility of the seized drug in evidence, it is necessary for us to
ascertain whether or not the search which yielded the alleged contraband
was lawful.16
The 1987 Constitution states that a search and consequent seizure must be
carried out with a judicial warrant; otherwise, it becomes unreasonable and
any evidence obtained therefrom shall be inadmissible for any purpose in any
proceeding.17 Said proscription, however, admits of exceptions, namely:
1. Warrantless search incidental to a lawful arrest;
2. Search of evidence in "plain view;"
3. Search of a moving vehicle;
4. Consented warrantless search;
5. Customs search;

6. Stop and Frisk; and


7. Exigent and emergency circumstances.18
What constitutes a reasonable or unreasonable warrantless search or seizure
is purely a judicial question, determinable from the uniqueness of the
circumstances involved, including the purpose of the search or seizure, the
presence or absence of probable cause, the manner in which the search and
seizure was made, the place or thing searched, and the character of the
articles procured.19
The RTC concluded that appellant was caught in flagrante delicto, declaring
that he was caught in the act of actually committing a crime or attempting to
commit a crime in the presence of the apprehending officers as he arrived in
Baler, Aurora bringing with him a sachet of shabu. 20 Consequently, the
warrantless search was considered valid as it was deemed an incident to the
lawful arrest.
Recent jurisprudence holds that in searches incident to a lawful arrest, the
arrest must precede the search; generally, the process cannot be reversed.
Nevertheless, a search substantially contemporaneous with an arrest can
precede the arrest if the police have probable cause to make the arrest at the
outset of the search. 21 Thus, given the factual milieu of the case, we have to
determine whether the police officers had probable cause to arrest appellant.
Although probable cause eludes exact and concrete definition, it ordinarily
signifies a reasonable ground of suspicion supported by circumstances
sufficiently strong in themselves to warrant a cautious man to believe that
the person accused is guilty of the offense with which he is charged. 22
The determination of the existence or absence of probable cause necessitates
a reexamination of the established facts. On May 19, 2003, a confidential
agent of the police transacted through cellular phone with appellant for the
purchase of shabu. The agent reported the transaction to the police
authorities who immediately formed a team to apprehend the appellant. On
May 20, 2003, at 11:00 a.m., appellant called up the agent with the
information that he was on board a Genesis bus and would arrive in Baler,
Aurora anytime of the day wearing a red and white striped T-shirt. The team
members posted themselves along the national highway in Baler, Aurora, and
at around 3:00 p.m. of the same day, a Genesis bus arrived in Baler. When
appellant alighted from the bus, the confidential agent pointed to him as the
person he transacted with, and when the latter was about to board a tricycle,
the team approached him and invited him to the police station as he was
suspected of carrying shabu. When he pulled out his hands from his pants
pocket, a white envelope slipped therefrom which, when opened, yielded a
small sachet containing the suspected drug. 23 The team then brought
appellant to the police station for investigation and the confiscated specimen
was marked in the presence of appellant. The field test and laboratory
examinations on the contents of the confiscated sachet yielded positive
results for methamphetamine hydrochloride.

Clearly, what prompted the police to apprehend appellant, even without a


warrant, was the tip given by the informant that appellant would arrive in
Baler, Aurora carrying shabu. This circumstance gives rise to another
question: whether that information, by itself, is sufficient probable cause to
effect a valid warrantless arrest.
The long standing rule in this jurisdiction is that "reliable information" alone
is not sufficient to justify a warrantless arrest. The rule requires, in addition,
that the accused perform some overt act that would indicate that he has
committed, is actually committing, or is attempting to commit an
offense.24 We find no cogent reason to depart from this well-established
doctrine.
The instant case is similar to People v. Aruta, 25 People v. Tudtud,26 and People
v. Nuevas.27
In People v. Aruta, a police officer was tipped off by his informant that a
certain "Aling Rosa" would be arriving from Baguio City the following day
with a large volume of marijuana. Acting on said tip, the police assembled a
team and deployed themselves near the Philippine National Bank (PNB) in
Olongapo City. While thus positioned, a Victory Liner Bus stopped in front of
the PNB building where two females and a man got off. The informant then
pointed to the team members the woman, "Aling Rosa," who was then
carrying a traveling bag. Thereafter, the team approached her and introduced
themselves. When asked about the contents of her bag, she handed it to the
apprehending officers. Upon inspection, the bag was found to contain dried
marijuana leaves.28
The facts in People v. Tudtud show that in July and August, 1999, the Toril
Police Station, Davao City, received a report from a civilian asset that the
neighbors of a certain Noel Tudtud (Tudtud) were complaining that the latter
was responsible for the proliferation of marijuana in the area. Reacting to the
report, the Intelligence Section conducted surveillance. For five days, they
gathered information and learned that Tudtud was involved in illegal drugs.
On August 1, 1999, the civilian asset informed the police that Tudtud had
headed to Cotabato and would be back later that day with a new stock of
marijuana. At around 4:00 p.m. that same day, a team of police officers
posted themselves to await Tudtuds arrival. At 8:00 p.m., two men
disembarked from a bus and helped each other carry a carton. The police
officers approached the suspects and asked if they could see the contents of
the box which yielded marijuana leaves.29
In People v. Nuevas, the police officers received information that a certain
male person, more or less 54" in height, 25 to 30 years old, with a tattoo
mark on the upper right hand, and usually wearing a sando and maong
pants, would make a delivery of marijuana leaves. While conducting
stationary surveillance and monitoring of illegal drug trafficking, they saw
the accused who fit the description, carrying a plastic bag. The police
accosted the accused and informed him that they were police officers. Upon

inspection of the plastic bag carried by the accused, the bag contained
marijuana dried leaves and bricks wrapped in a blue cloth. In his bid to
escape charges, the accused disclosed where two other male persons would
make a delivery of marijuana leaves. Upon seeing the two male persons, later
identified as Reynaldo Din and Fernando Inocencio, the police approached
them, introduced themselves as police officers, then inspected the bag they
were carrying. Upon inspection, the contents of the bag turned out to be
marijuana leaves.30
In all of these cases, we refused to validate the warrantless search precisely
because there was no adequate probable cause. We required the showing of
some overt act indicative of the criminal design.
As in the above cases, appellant herein was not committing a crime in the
presence of the police officers. Neither did the arresting officers have
personal knowledge of facts indicating that the person to be arrested had
committed, was committing, or about to commit an offense. At the time of
the arrest, appellant had just alighted from the Gemini bus and was waiting
for a tricycle. Appellant was not acting in any suspicious manner that would
engender a reasonable ground for the police officers to suspect and conclude
that he was committing or intending to commit a crime. Were it not for the
information given by the informant, appellant would not have been
apprehended and no search would have been made, and consequently, the
sachet of shabu would not have been confiscated.
We are not unaware of another set of jurisprudence that deems "reliable
information" sufficient to justify a search incident to a lawful warrantless
arrest. As cited in People v. Tudtud, these include People v.
Maspil,
Jr.,31 People
v.
Bagista,32 People
v.
Balingan,33 People
v.
34
35
36
Lising, People v. Montilla, People v. Valdez, and People v. Gonzales.37 In
these cases, the Court sustained the validity of the warrantless searches
notwithstanding the absence of overt acts or suspicious circumstances that
would indicate that the accused had committed, was actually committing, or
attempting to commit a crime. But as aptly observed by the Court, except in
Valdez and Gonzales, they were covered by the other exceptions to the rule
against warrantless searches.38
Neither were the arresting officers impelled by any urgency that would allow
them to do away with the requisite warrant. As testified to by Police Officer 1
Aurelio Iniwan, a member of the arresting team, their office received the
"tipped information" on May 19, 2003. They likewise learned from the
informant not only the appellants physical description but also his name.
Although it was not certain that appellant would arrive on the same day (May
19), there was an assurance that he would be there the following day (May
20). Clearly, the police had ample opportunity to apply for a warrant. 39
Obviously, this is an instance of seizure of the "fruit of the poisonous tree,"
hence, the confiscated item is inadmissible in evidence consonant with
Article III, Section 3(2) of the 1987 Constitution, "any evidence obtained in

violation of this or the preceding section shall be inadmissible for any


purpose in any proceeding."
Without the confiscated shabu, appellants conviction cannot be sustained
based on the remaining evidence. Thus, an acquittal is warranted, despite
the waiver of appellant of his right to question the illegality of his arrest by
entering a plea and his active participation in the trial of the case. As earlier
mentioned, the legality of an arrest affects only the jurisdiction of the court
over the person of the accused. A waiver of an illegal, warrantless arrest does
not carry with it a waiver of the inadmissibility of evidence seized during an
illegal warrantless arrest.40
One final note. As clearly stated in People v. Nuevas,41
x x x In the final analysis, we in the administration of justice would have no
right to expect ordinary people to be law-abiding if we do not insist on the full
protection of their rights. Some lawmen, prosecutors and judges may still
tend to gloss over an illegal search and seizure as long as the law enforcers
show the alleged evidence of the crime regardless of the methods by which
they were obtained. This kind of attitude condones law-breaking in the name
of law enforcement. Ironically, it only fosters the more rapid breakdown of our
system of justice, and the eventual denigration of society. While this Court
appreciates and encourages the efforts of law enforcers to uphold the law and
to preserve the peace and security of society, we nevertheless admonish them
to act with deliberate care and within the parameters set by the Constitution
and the law. Truly, the end never justifies the means. 42
WHEREFORE, premises considered, the Court of Appeals Decision dated
May 22, 2008 in CA-G.R. CR-H.C. No. 00425 is REVERSED and SET ASIDE.
Appellant Jack Raquero Racho is ACQUITTED for insufficiency of evidence.
The Director of the Bureau of Corrections is directed to cause the immediate
release of appellant, unless the latter is being lawfully held for another cause;
and to inform the Court of the date of his release, or the reasons for his
confinement, within ten (10) days from notice.
No costs.
C. General or roving warrants
1. Harry Stonehill et al vs DOJ Secretary Jose Diokno et al
Search and Seizure General Warrants Abandonment of the Moncado
Doctrine
Stonehill et al and the corporation they form were alleged to have committed
acts in violation of Central Bank Laws, Tariff and Customs Laws, Internal
Revenue (Code) and Revised Penal Code. By the strength of this allegation a
search warrant was issued against their persons and their corporation. The
warrant provides authority to search the persons above-named and/or the
premises of their offices, warehouses and/or residences, and to seize and
take possession of the following personal property to wit:

Books of accounts, financial records, vouchers, correspondence, receipts,


ledgers, journals, portfolios, credit journals, typewriters, and other
documents and/or papers showing all business transactions including
disbursements receipts, balance sheets and profit and loss statements and
Bobbins (cigarette wrappers).
The documents, papers, and things seized under the alleged authority of the
warrants in question may be split into (2) major groups, namely:
(a) those found and seized in the offices of the aforementioned corporations
and
(b) those found seized in the residences of petitioners herein.
Stonehill averred that the warrant is illegal for:
(1) they do not describe with particularity the documents, books and things
to be seized;
(2) cash money, not mentioned in the warrants, were actually seized;
(3) the warrants were issued to fish evidence against the aforementioned
petitioners in deportation cases filed against them;
(4) the searches and seizures were made in an illegal manner; and
(5) the documents, papers and cash money seized were not delivered to the
courts that issued the warrants, to be disposed of in accordance with law.
The prosecution counters, invoking the Moncado doctrine, that the defects of
said warrants, if any, were cured by petitioners consent; and (3) that, in any
event, the effects seized are admissible in evidence against them. In short,
the criminal cannot be set free just because the government blunders.
ISSUE: Whether or not the search warrant issued is valid.
HELD: The SC ruled in favor of Stonehill et al. The SC emphasized however
that Stonehill et al cannot assail the validity of the search warrant issued
against their corporation for Stonehill are not the proper party hence has no
cause of action. It should be raised by the officers or board members of the
corporation. The constitution protects the peoples right against
unreasonable search and seizure. It provides; (1) that no warrant shall issue
but upon probable cause, to be determined by the judge in the manner set
forth in said provision; and (2) that the warrant shall particularly describe
the things to be seized. In the case at bar, none of these are met. The warrant
was issued from mere allegation that Stonehill et al committed a violation of
Central Bank Laws, Tariff and Customs Laws, Internal Revenue (Code) and
Revised Penal Code. In other words, no specific offense had been alleged in
said applications. The averments thereof with respect to the offense
committed were abstract. As a consequence, it was impossible for the judges
who issued the warrants to have found the existence of probable cause, for
the same presupposes the introduction of competent proof that the party
against whom it is sought has performed particular acts, or committed
specific omissions, violating a given provision of our criminal laws. As a
matter of fact, the applications involved in this case do not allege any specific
acts performed by herein petitioners. It would be a legal heresy, of the highest

order, to convict anybody of a violation of Central Bank Laws, Tariff and


Customs Laws, Internal Revenue (Code) and Revised Penal Code, as
alleged in the aforementioned applications without reference to any
determinate provision of said laws or codes.
The grave violation of the Constitution made in the application for the
contested search warrants was compounded by the description therein made
of the effects to be searched for and seized, to wit:
Books of accounts, financial records, vouchers, journals, correspondence,
receipts, ledgers, portfolios, credit journals, typewriters, and other
documents and/or papers showing all business transactions including
disbursement receipts, balance sheets and related profit and loss
statements.
Thus, the warrants authorized the search for and seizure of records
pertaining to all business transactions of Stonehill et al, regardless of
whether the transactions were legal or illegal. The warrants sanctioned the
seizure of all records of Stonehill et al and the aforementioned corporations,
whatever their nature, thus openly contravening the explicit command of the
Bill of Rights that the things to be seized be particularly described as
well as tending to defeat its major objective: the elimination of general
warrants. The Moncado doctrine is likewise abandoned and the right of the
accused against a defective search warrant is emphasized.
2. Bache & Co. Inc. et al vs BIR Commissioner Vivencio Ruiz et al
Search and Seizure Personal Examination of the Judge
On 24 Feb 1970, Commissioner Vera of Internal Revenue, wrote a letter
addressed to J Ruiz requesting the issuance of a search warrant against
petitioners for violation of Sec 46(a) of the NIRC, in relation to all other
pertinent provisions thereof, particularly Sects 53, 72, 73, 208 and 209, and
authorizing Revenue Examiner de Leon make and file the application for
search warrant which was attached to the letter. The next day, de Leon and
his witnesses went to CFI Rizal to obtain the search warrant. At that time J
Ruiz was hearing a certain case; so, by means of a note, he instructed his
Deputy Clerk of Court to take the depositions of De Leon and Logronio. After
the session had adjourned, J Ruiz was informed that the depositions had
already been taken. The stenographer read to him her stenographic notes;
and thereafter, J Ruiz asked respondent Logronio to take the oath and
warned him that if his deposition was found to be false and without legal
basis, he could be charged for perjury. J Ruiz signed de Leons application
for search warrant and Logronios deposition. The search was subsequently
conducted.
ISSUE: Whether or not there had been a valid search warrant.
HELD: The SC ruled in favor of Bache on three grounds.
1. J Ruiz failed to personally examine the complainant and his witness.

Personal examination by the judge of the complainant and his witnesses is


necessary to enable him to determine the existence or non-existence of a
probable cause.
2. The search warrant was issued for more than one specific offense.
The search warrant in question was issued for at least four distinct offenses
under the Tax Code. As ruled in Stonehill Such is the seriousness of the
irregularities committed in connection with the disputed search warrants,
that this Court deemed it fit to amend Section 3 of Rule 122 of the former
Rules of Court that a search warrant shall not issue but upon probable
cause in connection with one specific offense. Not satisfied with this
qualification, the Court added thereto a paragraph, directing that no search
warrant shall issue for more than one specific offense.
3. The search warrant does not particularly describe the things to be seized.
The documents, papers and effects sought to be seized are described in the
Search Warrant
Unregistered and private books of accounts (ledgers, journals, columnars,
receipts and disbursements books, customers ledgers); receipts for payments
received; certificates of stocks and securities; contracts, promissory notes
and deeds of sale; telex and coded messages; business communications,
accounting and business records; checks and check stubs; records of bank
deposits and withdrawals; and records of foreign remittances, covering the
years 1966 to 1970.
The description does not meet the requirement in Art III, Sec. 1, of the
Constitution, and of Sec. 3, Rule 126 of the Revised Rules of Court, that the
warrant should particularly describe the things to be seized.
A search warrant may be said to particularly describe the things to be seized
when the description therein is as specific as the circumstances will
ordinarily allow or when the description expresses a conclusion of fact not of
law by which the warrant officer may be guided in making the search and
seizure or when the things described are limited to those which bear direct
relation to the offense for which the warrant is being issued.
3. DOJ Sec Vicente Abad Santos vs CFI Benguet Judge Pio Marcos
Search and Seizure
On March 31, 1971, Amansec went to Baguio and passed by a house at 47
Ledesma Street, Baguio; he was attracted by the sight of several persons
inside the house; he peeped from outside the house and when the curtain
was moved he saw a Buddha that was inside the house; he observed what
was going on inside the house and he heard someone say that the golden
Buddha was actually for sale and when he observed them closer he
overheard that it was being offered for sale for 100,000 pesos by Rogelio
Roxas; he saw the Buddha and firearms and some bullets inside the house.
By these facts, Colonel Calano requested for a warrant from J Marcos at
about 12 midnight on Apr 4, 1971. Due to the urgency he issued the

warrant. And eventually the golden Buddha and some firearms were seized
from Roxass house. Santos assailed the warrant averring that the search
warrant was not limited to one offense covering both illegal possession of
firearms and violation of Central Bank rules and regulations; that it did not
particularly describe the property to be seized; that he did not carefully
examine under oath the applicant and his witnesses; that articles not
mentioned were taken; and that thereafter the return and the inventory
although appearing to have been prepared on said date were not actually
submitted to respondent Judge until April 13, 1971 and the objects seized
delivered only about a week later on April 19.
ISSUE: Whether or not the search warrant issued by Judge Marcos is valid.
HELD: The SC ruled in favor Judge Marcos and had basically affirmed the
decision of appellate Judge Gatamaitan. Taking into consideration to nature
of the articles so described, it is clear that no other more adequate and
detailed description could be given, particularly because it is difficult to give
a particular description of the contents thereof, The description so made
substantially complies with the legal provisions because the officer of the law
who executed the warrant was thereby placed in a position enabling him to
Identify the articles in question, which he did, so that here, since
certainly, no one would be mistaken in Identifying the Buddha, whose image
is well known, and even the firearms and ammunition because these were
those without permit to possess, and all located at 47 Ledesma St., Baguio
City, so far as description was concerned, the search warrant perhaps could
not be said to have suffered fatal defects.
4. Castro v Pabalan
This Court is confronted anew in this certiorari proceeding with the claim
that a search warrant issued without complying with the requisites of the
Constitution 1 and the Rules of Court 2 should have been nullified, but was
not in the challenged order of respondent Judge Javier Pabalan. 3 More
specifically, it was the failure of the application for the search warrant as well
as the search warrant itself to specify the specific offense, to examine the
applicant as well as his witnesses on the part of respondent Judge, and to
describe with particularity the place to be searched and the things to be
seized, that were singled out to justify the assertion of illegality. When
required to answer, respondent Judge did not bother to refute specifically the
allegations of the petition for certiorari, but merely contented himself with
inviting attention to the challenged order as well as the resolutions denying
the motion for reconsideration and with the statement that he "has no
particular prayer to ask the Supreme Court," an assertion thereafter repeated
in the second paragraph of his two-paragraph answer that he "has no
request to make in this particular case leaving the issues entirely to the
discretion of the Supreme Court." 4 The tone of diffidence, almost of apology,
is easy to understand. It is difficult to resist the thought that respondent

Judge failed to pay heed to authoritative decisions of this Court. The most
cursory perusal of the application for search warrant 5 by respondent
Lumang and the search warrant itself, 6 yields no other conclusion.
Respondent Judge ignored what the Constitution requires on two points, the
existence of a probable cause and the particular description of the things to
be seized. The limitation as to the specific offense as mandated by the Rules
of Court was not observed either. Even on the assumption then that he could
not
relevant According to the former: "A search warrant shall not issue but upon
probable cause in connection with one specific offense to be determined by
the municipal or city judge after examination under oath or affirmation of
the complainant and the witnesses he may produce, and particularly
describing the place to be searched and the persons or things to be seized.
No search warrant shall issue for more than one specific offense." Section 4
provides: "The municipal or city judge must, before issuing the warrant,
personally examine on oath or affirmation the complainant and any
witnesses he may produce and take their depositions in writing, and attach
them to the record, in addition to any affidavits presented to him." be held
chargeable with knowledge of the leading Stonehill decision, 7 announced
barely twenty days before the search warrant in question was issued, still
from Alverez v. Court of First Instance 8 the first to be decided under the
1935 Constitution, promulgated in 1937, to Oca v. Marquez, 9 that came out
in 1965, this Court had adhered firmly to the view that for a search warrant
to escape the imputation of being unreasonable, there should be strict
conformity with the requirements of the Constitution and the applicable
procedural rules. The finding then should have been against the validity of
the search warrant. Nonetheless, insofar as such order limited itself to
requiring the return solely of the liquor, the pack of playing cards, the bottle
of distilled water and five bottles of Streptomycin, all of which may be
considered as personal effects of petitioners, with the rest of the goods taken
falling under the category of things forbidden by law and therefore need not
be restored, 10 it can be sustained. So we rule.
In the opening paragraph of the application for search warrant, respondent
Ernesto I. Lumang admitted that "he has been informed" and therefore was of
the belief that petitioners Maria Castro and Co Ling, whose place of residence
was not even indicated, although subsequently mention was made of their
being at Barrio Padasil, Bangar, La Union, "have in possession narcotics and
other contraband." 11 There is a claim that he had verified the report and that
therefore he had "reasons to believe that a Search Warrant should be issued
to enable the undersigned to take possession" of such narcotics and other
contraband. 12 The application was accompanied by the joint affidavit of a
Sergeant Francisco C. Molina and a Corporal Lorenzo G. Apilado of the
Philippine Constabulary. 13 Again, mention was merely made of their

information about narcotics and other contraband being kept by petitioners.


They did allege therein that they conducted rigid surveillance, but all they
could come out with is that petitioner Co Ling is an overstaying alien for
almost ten years conducting such traffic and that after verification, he was
not registered in the Immigration Office. 14 Then, on the very same day, July
10, 1967, the search warrant was issued for illegal traffic of narcotics and
contraband. 15 Again, there was reference to the possession by petitioners of
such forbidden goods. As to the complete and detailed description of the
properties to be seized, the search warrant merely mentioned illegal traffic of
narcotics and contraband inside the warehouse and premises of
petitioners. 16 In the resolution upholding the validity of the search warrant,
respondent Judge did state the following: "On July 10, 1967, Ernesto
Lumang, Sgt. of the PC, with a long service behind, appeared in chamber
before the Presiding Judge of Branch I of this Court. With him were Sgt.
Molina and Cpl. Apilado both of the PC Command of La Union. The three
submitted to the Presiding Judge in chamber an application for search
warrant which is Exhibit I in this case and a joint affidavit supporting the
search warrant asked. As Sgt. Lumang said, testifying regarding this
incident, those appearing were asked, although not in writing and not
recorded, some questions by the Presiding Judge regarding their request of
the search warrant on the knowledge of Molina and Apilado on the facts
stated on the application and on the joint affidavit. The inquiry was brief.
The barrio to be searched was handwritten in ink, Maria Cristina cancelling
the typewritten name Padasil. But this correction was not done in the
duplicates. Anyhow Padasil and Maria Cristina are adjoining barrios. After
the routine taking of their oath and examination questions and answers, the
Presiding Judge of this Branch signed the application for search warrant, the
joint affidavits, and forthwith issued the search warrant which is Exhibit
C." 17
As set forth at the outset, failure to abide by both the Constitution and the
procedural law in terms of the existence of a probable cause, a particular
description of the property to be seized and the requirement that there be
only one specific offense, is quite manifest.
1. This excerpt from the epochal opinion of former Chief Justice Concepcion
in Stonehill v. Diokno 18 is highly relevant: "Two points must be stressed in
connection with this constitutional mandate, namely: (1) that no warrant
shall issue but upon probable cause, to be determined by the judge in the
manner set forth in said provision; and (2) that the warrant
shall particularly describe the things to be seized. None of these requirements
has been complied with in the contested warrants. Indeed, the same were
issued upon applications stating that the natural and juridical persons
therein named had committed a 'violation of Central Bank Laws, Tariff and
Customs Laws, Internal Revenue (Code) and Revised Penal Code.' In other
words, no specific offense had been alleged in said applications. The

averments thereof with respect to the offense committed were abstract. As a


consequence, it wasimpossible for the judges who issued the warrants to
have found the existence of probable cause, for the same presupposes the
introduction of competent proof that the party against whom it is sought has
performed particularacts, or committed specific omissions, violating a given
provision of our criminal laws. As a matter of fact, the applications involved
in this case do not allege any specific acts performed by herein petitioners. It
would be a legal heresy, of the highest order, to convict anybody of a 'violation
of Central Bank Laws, Tariff and Customs Laws, Internal Revenue (Code) and
Revised Penal Code,' as alleged in the aforementioned applications
without reference to any determinate provision of said laws or codes." 19 That
same approach is reflected in the two subsequent cases of Bache & Co. (Phil.),
Inc. v. Ruiz 20 and Asian Surety & Insurance Co., Inc. v. Herrera. 21 It bears
repeating, as was emphasized in Stonehill v. Diokno, that the averments as to
the alleged commission of the offenses imputed to petitioner were abstract.
As admitted in the challenged order, the inquiry was brief. Subsequently,
reference was made to "the routine taking of [their oath] and examination
questions and answers ..."22 Nor can such perfunctory manner in which
respondent Judge conducted the required "examination under oath" be
justified merely because respondent Lumang was "a Sergeant of the PC, with
a long service behind [him]." 23Moreover, contrary to the Rules of Court, he
did not even bother to take the depositions of the witnesses in writing,
attaching them to the record. 24 There was thus a manifest and palpable
violation of the constitutional standard as to the quantum of proof to show
the existence of probable cause, as so clearly enunciated in Stonehill.
2. Then again, the Constitution requires, for the validity of a search warrant,
that there be a particular description of "the place to be searched and the
persons or things to be seized." 25 As was admitted by the judge in the
challenged resolution, there was a mistake concerning the residence of
petitioners, which was set forth in the search warrant as being in Barrio
Padasil when in fact it is in Barrio Maria Cristina. He would gloss over such
inaccuracy by saying that they were, anyway, adjoining barrios. As to the
premises to be searched, it may be admitted that the deficiency in the writ is
not of sufficient gravity to call for its invalidation. Nonetheless, and again in
line with Stonehill v. Diokno, the Constitution is quite explicit that there be a
particular description of the things to be seized. That requisite was not
complied with in this case. That would explain why the searching party felt it
had a free hand and did take possession of various kinds of goods, including
personal effects, which respondent Judge himself would have them return.
What was aptly characterized as a "major objective" of this constitutional
provision, the elimination of general warrants, was thus frustrated. It need
not be stressed anew that this Court is resolutely committed to the doctrine
that this constitutional provision is of a mandatory character and therefore
must be strictly complied with. 26 To quote from the landmark American

decision of Boyd v. United States: 27 "It is the duty of courts to be watchful for
the constitutional rights of the citizen, and against any stealthy
encroachments thereon. Their motto should be obsta principis." 28
3. Another infirmity was the failure to comply with the basic procedural
requisite that a search warrant "shall not issue but upon probable cause in
connection with one specific offense." 29 Here reference was made to "an
illegal traffic of narcotics and contraband." The latter is a generic term
covering all goods exported from or imported into the country contrary to
applicable statutes. Necessarily then, more than one offense could arise from
the activity designated as illegal traffic of narcotics and contraband. As a
matter of fact, in the challenged order, reference was made to at least three
charges having been filed, the violation of Section 203 of the Internal
Revenue Code, its Section 1039 on tax evasion, as well as illegal possession
of opium. It would seem that once again what was correctly pointed out
by Chief Justice Concepcion in Stonehill v. Diokno as unjustified and
unwarranted finds application. Nor can there be any plausibility to the
possible excuse, to repeat what was said before, that the Stonehill opinion
having been rendered only twenty days previous to the issuance of the search
warrant, respondent Judge could not be held chargeable with a knowledge
thereof, considering that as far back as July 30, 1965, two years earlier,
in Oca v. Marquez, 30 this Court, through the then Justice J. P. Bengzon,
enunciated: "The decision herein has applied the provisions of th Old Rules
of Court since this case arose under said Rules. Attention of the Bench and
Bar is however called to the fact that effective January 1, 1964 the issuance
of search warrants is governed by Section 3, Rule 126 of the Revised Rules of
Court which among other things requires that a search warrant must be in
connection with one specific offense." 31
4. As was made clear at the outset, though, the illegality of the search
warrant does not call for the return of the things seized, the possession of
which is prohibited by law. This is the established doctrine in this
jurisdiction. As far back as Uy Kheytin v. Villareal, 32 a 1920 decision, it was
held: "That although in the issuance of the search warrant in question the
judge did not comply with the requirements of section 98 of General Orders
No. 58, the petitioners are not entitled to the return of the opium. and its
paraphernalia which were found and seized under said warrant, and much
less are they entitled to be exonerated because of such omission of the
judge." 33 Among the authorities cited is Cooley: "'Search-warrants have
heretofore been allowed to search for stolen goods, for goods supposed to
have been smuggled into the country in violation of the revenue laws, for
implements of gaming or counterfeiting, for lottery tickets or Prohibited
liquors kept for sale contrary to law, for obscene books and papers kept for
sale or circulation, and for powder or other explosive and dangerous material
so kept as to endanger the public safety.'" 34 So, also, in Yee Sue Koy v.

Almeda, 35 handed down in 1940, Justice Laurel, speaking for this Court,
stated: "If it be true, furthermore, without, however, deciding the point, that
as alleged by the respondents the articles in question constitute the corpus
delicti of the Usury Law, their return to the petitioners cannot be
ordered."36 Magoncia v. Palacios, 37 promulgated in 1948, reiterated such a
doctrine. Thus: "En el asunto de Uy Kheytincontra Villareal (42 Jur. Fil. 935),
los recurrentes pidieron la devolucion del opio de que se incautaron los
constabularies al registrar su casa armados con un mandamiento de registro
expedido sin cumplir las disposiciones de los articulos 96 y 98 de la Orden
General No. 58; sostenian que los requisites exigidos por dichos articulos no
se habian cumplido, y por tanto, el mandamiento de registro era ilegal, como
si no existiera; que al registro se ha hecho sin mandamiento de registro
debidamente expedido. Este Tribunal denego la peticion, declarando que la
irregularidad de la expedicion del mandamiento de registro ne era suficiente
causa para ordenar la devolucion del opio. El Hon. Juez recurrido no abuso
de su discrecion al denegar la devolucion al acusado del paltik, 42
municiones y una granada de mano, tampoco abuso de su sana discrecion al
denegar la peticion del acusado de que se prohiba al Fiscal Provincial y al
Jefe de Policia de Asingan, Pangasinan a presentar tales efectos como prueba
en la vista." 38
5. This decision leaves open the question of the legality of any possible use
that may be made by the prosecuting authorities of the articles seized under
an invalid search warrant. Here, again, the Yee Sue Koy opinion of Justice
Laurel is illuminating, especially in view of the inadmissibility of evidence
illegally seized under the present Constitution 39 At this stage, the question
does not have to be faced. The words of Justice Laurel follow: "While we
reiterate the rule that the seizure of books and documents by means of a
search warrant ' for the purpose of using them as evidence in a criminal case
against the person in whose possession they were found is unconstitutional
because it makes the warrant unreasonable, and it is equivalent to a
violation of the constitutional provision prohibiting the compulsion of an
accused to testify against himself ..., the said rule has no applicable force in
the present case. ... In the application for the issuance of the search warrant
in question, it was alleged that the articles seized were 'being used by it (Sam
Sing & Co.) in connection with its activities of lending money at usurious rate
of interest in violation of the Usury Law,' and it is now suggested (memoranda
of respondents) that the only object of the agents of the Anti-Usury Board in
keeping the articles is to prevent the petitioners from employing them as a
means of further violations of the Usury Law. In this state of the record,
without deciding the question whether the petitioners will in fact use the
articles in question, if returned, for illegal purposes, we are not prepared to
order the return prayed for by the petitioners. (Cf. People v. Rubio, 57 Phil.
384, 394-395.)" 40

WHEREFORE, the writ of certiorari is granted and the order of September 12,
1967 denying the motion of petitioners to annul the search warrant as well
as the resolutions of October 26, 1967 and January 29, 1968 denying the
motions for reconsiderations are reversed, the decision of this Court being
that the search warrant in question is tainted by illegality for being violative
both of the Constitution and the Rules of Court. It is likewise the decision of
this Court that notwithstanding the illegality of such search warrant, the
challenged order of respondent Judge can be sustained only insofar as it
would limit the return of the articles seized to the liquor, the pack of playing
cards, the bottle of distilled water and five bottles of Streptomycin taken
under such search warrant. No costs.
Barred, Antonio, Aquino and Concepcion, Jr., JJ., concur.
5. Asian Surety vs Herrera
Petition to quash and annul a search warrant issued by respondent Judge
Jose Herrera of the City Court of Manila, and to command respondents to
return immediately the documents, papers, receipts and records alleged to
have been illegally seized thereunder by agents of the National Bureau of
Investigation (NBI) led by respondent Celso Zoleta, Jr.
On October 27, 1965, respondent Judge Herrera, upon the sworn application
of NBI agent Celso Zoleta, Jr. supported by the deposition of his witness,
Manuel Cuaresma, issued a search warrant in connection with an
undocketed criminal case for estafa, falsification, insurance fraud, and tax
evasion, against the Asian Surety and Insurance Co., a corporation duly
organized and existing under the laws of the Philippines, with principal office
at Room 200 Republic Supermarket Bldg., Rizal Avenue, Manila. The search
warrant is couched in the following language:
It appearing to the satisfaction of the undersigned, after examining under
oath NBI Agent Celso J. Zoleta, Jr. and his witness Manuel Cuaresma that
there are good and sufficient reasons to believe thatMr. William Li Yao or his
employees has/have in his/their control in premises No. 2nd Floor Republic
Supermarket Building, in Rizal Avenue district of Sta. Cruz, Manila, property
(Subject of the offense; stolen or embezzled and proceeds or fruits of the
offense used or intended to be used as the means of committing the offense)
should be seized and brought to the undersigned.
You are hereby commanded to make an immediate search at any time in the
----- of the premises above-described and forthwith seize and take possession
of the following personal property to wit: Fire Registers, Loss Bordereau,
Adjusters Report including subrogation receipt and proof of loss, Loss
Registers, Books of Accounts, including cash receipts and disbursements and
general ledger, check vouchers, income tax returns, and other
papers connected therewith ... for the years 1961 to 1964 to be dealt with as
the law directs.

Armed with the search warrant Zoleta and other agents assigned to the Antigraft Division of the NBI entered the premises of the Republic Supermarket
Building and served the search warrant upon Atty. Alidio of the insurance
company, in the presence of Mr. William Li Yao, president and chairman of
the board of directors of the insurance firm. After the search they seized and
carried away two (2) carloads of documents, papers and receipts.
Petitioner assails the validity of the search warrant, claiming that it was
issued in contravention of the explicit provisions of the Constitution and the
Rules of Court, particularly Section 1, of Art. III of the 1935 Constitution,
now Section 3, of Art. IV of the new Constitution, and Sections 3, 5, 8 and 10
of Rule 126 of the Rules of Court, hereunder quoted for convenience of
reference, viz:
Sec. 3 The rights of the people to be secure in their persons, houses,
papers and effects, against unreasonable searches and seizures shall not be
violated, and no warrant shall issue but upon probable cause to be
determined by the judge after examination under oath or affirmation of the
complainant and the witnessed he may produce, and particularly describing
the place to be searched, and the persons, or things to be seized." (Art. IV,
Section 3, New Constitution)
Sec. 3 Requisites for issuing search warrant A search warrant shall not
issue but upon probable cause in connection with one specific offense to be
determined by the judge or justice of the peace after examination under oath
or affirmation of the complainant and the witnesses he may produce, and
particularly describing the place to be searched and the persons or things to
be seized.
No search warrant shall issue for more than one specific offense. (Sec. 3,
Rule 126, Rules of Court)
Sec. 5 Issuance and form of search warrant If the judge or justice of the
peace is thereupon satisfied of the existence of facts upon which the
application is based, or that there is probable cause to believe that they exist,
he must issue the warrant in the form prescribed by these rules. (Sec. 5,
Rule 126)
Sec. 8 Time of making search The warrant must direct that it be served
in the day time, unless the affidavit asserts that the property is on the
person or in the place ordered to be searched, in which case a direction may
be inserted that it be served at any time of the night or day. (Sec. 8, Rule
126)
Sec. 10 Receipt for property seized. The officer seizing property under the
warrant must give a detailed receipt for the same to the person on whom or
in whose possession it was found, or in the absence of any person, must, in
the presence of at least one witness, leave a receipt in the place in which he
found the seized property. (Sec. 10, Rule 126) .
"Of all the rights of a citizen, few are of greater importance or more essential
to his peace and happiness than the right of personal security, and that

involves the exemption of his private affairs, books, and papers from the
inspection and scrutiny of others. 1 While the power to search and seize is
necessary to the public welfare, still it must be exercised and the law
enforced without transgressing the constitutional rights of the citizens, for
the enforcement of no statute is of sufficient importance to justify
indifference to the basic principles of government (People v. Elias, 147 N.E.
472)."
I.
In the case at bar, the search warrant was issued for four separate and
distinct offenses of : (1) estafa, (2) falsification, (3) tax evasion and (4)
insurance fraud, in contravention of the explicit command of Section 3, Rule
126, of the Rules providing that: "no search warrant shall issue for more
than one specific offense." The aforequoted provision, which is found in the
last paragraph of the same section, is something new. "There is no precedent
on this amendment prohibition against the issuance of a search warrant
for more than one specific offense either in the American books on
Criminal procedure or in American decisions." 2 It was applied in the
celebrated case of Harry S. Stonehill v. Secretary of Justice 3 where this Court
said:
To uphold the validity of the warrants in question would be to wipe out
completely one of the most fundamental rights guaranteed in our
Constitution, for it would place the sanctity of the domicile and the privacy of
communication and correspondence at the mercy of the whims, caprice or
passion of peace officers. This is precisely the evil sought to be remedied by
the constitutional provision abovequoted to outlaw the so-called general
warrants. It is not difficult to imagine what would happen in times of keen
political strife, when the party in power feels that the minority is likely to
wrest it, even though by legal means.
Such is the seriousness of the irregularities committed in connection with
the disputed search warrants, that this Court deemed it fit to amend section
3 of Rule 122 of the former Rules of Court by providing in its counterpart,
under the Revised Rules of Court, that a search warrant shall not issue but
upon probable cause in connection with one specific offense. Not satisfied
with this qualification, the court added thereto a paragraph, directing that no
search warrant shall issue for more than one specific offense.
II.
Petitioner likewise contests the validity of the search warrant on the ground
that it authorized the search and seizures of personal properties so vaguely
described and not particularized, thereby infringing the constitutional
mandate requiring particular description of the place to be searched and the
persons or things to be seized. It also assails the noncompliance with the
above-requirement as likewise openly violative of Section 2 of Rule 126 which
provides:

SEC. 2. A search warrant may be issued for the search and seizure of the
following personal property:
(a) Property subject of the offense;
(b) Property stolen or embezzled and other proceeds or fruits of the offense;
and
(c) Property used or intended to be used as the means of committing an
offense.
The search warrant herein involved reads in part: "... property (Subject of the
offense, stolen or embezzled and proceeds or fruits of the offense used or
intended to be used as the means of committing the offense) should be seized
and brought to the undersigned." The claim of respondents that by not
cancelling the description of one or two of the classes of property contained
in the form when not applicable to the properties sought to be seized, the
respondent judge intended the search to apply to all the three classes of
property. This is a patent impossibility because the description of the
property to be searched and seized, viz: Fire Registers, Loss Bordereau,
Adjusters Report, including subrogation receipts and proof of loss, Loss
Registers, Books of Accounts including cash receipts and disbursements and
general ledger, etc. and the offenses alleged to have been committed by the
corporation to wit: estafa, falsification, tax evasion and insurance fraud,
render it impossible for Us to see how the above-described property can
simultaneously be contraband goods, stolen or embezzled and other proceeds
or fruits of one and the same offense. What is plain and clear is the fact that
the respondent Judge made no attempt to determine whether the property he
authorized to be searched and seized pertains specifically to any one of the
three classes of personal property that may be searched and seized under a
search warrant under Rule 126, Sec. 2 of the Rules. The respondent Judge
simply authorized search and seizure under an omnibus description of the
personal properties to be seized. Because of this all embracing description
which includes all conceivable records of petitioner corporation, which if
seized (as it was really seized in the case at bar), could possibly paralyze its
business, 4 petitioner in several motions, filed for early resolution of this case,
manifested that the seizure of TWO carloads of their papers has paralyzed
their business to the grave prejudice of not only the company, its workers,
agents, employees but also of its numerous insured and beneficiaries of
bonds issued by it, including the government itself, and of the general
public. 5 And correlating the same to the charges for which the warrant was
issued, We have before Us the infamous general warrants of old. In the case
of Uy Kheytin, et al., v. Villareal, 42 Phil. 896, cited with approval in the
Bache case, supra, We had occasion to explain the purpose of the
requirement that the warrant should particularly describe the place to be
searched and the things to be seized, to wit:
"... Both the Jones Law (sec. 3) and General Orders No. 58 (sec. 97)
specifically require that a search warrant should particularly describe the

place to be searched and the things to be seized. The evident purpose and
intent of this requirement is to limit the things to be seized to those, and only
those, particularly described in the search warrant to leave the officers of
the law with no discretion regarding what articles they shall seize, to the end
that "unreasonable searches and seizures" may not be made. That this is the
correct interpretation of this constitutional provision is borne out by
American authorities."
The purpose as thus explained could, surely and effectively, be defeated
under the search warrant issued in this case.
III.
Moreover, as contended by petitioner, respondents in like manner
transgressed Section 10 of Rule 126 of the Rules for failure to give a detailed
receipt of the things seized. Going over the receipts (Annexes "B", "B-1", B-2",
"B-3" and "B-4" of the Petition) issued, We found the following: one bordereau
of reinsurance, 8 fire registers, 1 marine register, four annual statements,
folders described only as Bundle gm-1 red folders; bundle 17-22 big carton
folders; folders of various sizes, etc., without stating therein the nature and
kind of documents contained in the folders of which there were about a
thousand of them that were seized. In the seizure of two carloads of
documents and other papers, the possibility that the respondents took away
private papers of the petitioner, in violation of his constitutional rights, is not
remote, for the NBI agents virtually had a field day with the broad and
unlimited search warrant issued by respondent Judge as their passport.
IV.
The search warrant violated the specific injunctions of Section 8 of Rule
126. 6 Annex "A" of the Petition which is the search warrant in question left
blank the "time" for making search, while actual search was conducted in the
evening of October 27, 1965, at 7:30 p.m., until the wee hours of the
morning of October 28, 1965, thus causing untold inconveniences to
petitioners herein. Authorities 7 are of the view that where a search is to be
made during the night time, the authority for executing the same at that
time should appear in the directive on the face of the warrant.
In their Memorandum 8 respondents, relying on the case of Moncado v.
Peoples Court (80 Phil. 1), argued:
Even assuming that the search warrant in question is null and void, the
illegality thereof would not render the incriminating documents inadmissible
in evidence.
This Court has reverted to the old rule and abandoned the Moncado ruling
(Stonehill case, supra). Most common law jurisdictions have already given up
this approach and eventually adopted the exclusionary rule, realizing that
this is the only practical means of enforcing the constitutional injunction
against unreasonable searches and seizures. Thus the Supreme Court of the
United States declared: 9

If letters and private documents can thus be seized and held and used in
evidence against a citizen accused of an offense the protection of the 4th
Amendment, declaring his right to be secured against such searches and
seizures is of no value, and so far as those thus placed are concerned, might
as well be stricken from the Constitution. The efforts of the courts and their
officials to bring the guilty to punishment, praise-worthy as they are, are not
to be aided by the sacrifice of those great principles established by years of
endeavor and suffering which have resulted in their embodiment in the
fundamental law of the land.
Moreover, the criminal charges filed by the NBI have all been dismissed
and/or dropped by the Court or by the office of the City Fiscal of Manila in
1968, as manifested in the petition filed by petitioner dated October 24,
1972, for early resolution of this case.
V.
It has likewise been observed that the offenses alleged took place from 1961
to 1964, and the application for search warrant was made on October 27,
1965. The time of the application is so far remote in time as to make the
probable cause of doubtful veracity and the warrant vitally defective. Thus
Mr. Joseph Varon, an eminent authority on Searches, Seizures and
Immunities, has this to say on this point:
From the examination of the several cases touching upon this subject, the
following general rules are said to apply to affidavits for search warrants:
(1) xxx xxx xxx
(2) Such statement as to the time of the alleged offense must be clear and
definite and must not be too remote from the time of the making of the affidavit
and issuance of the search warrant.
(3) There is no rigid rule for determining whether the stated time of
observation of the offense is too remote from the time when the affidavit is
made or the search warrant issued, but, generally speaking, a lapse of time
of more than three weeks will be held not to invalidate the search warrant
while a lapse of four weeks will be held to be so.
A good and practical rule of thumb to measure the nearness of time given in
the affidavit as to the date of the alleged offense, and the time of making the
affidavit is thus expressed: The nearer the time at which the observation of the
offense is alleged to have been made, the more reasonable the conclusion of
establishment of probable cause. [Emphasis Ours]
PREMISES CONSIDERED, petition is hereby granted; the search warrant of
October 27, 1965, is nullified and set aside, and the respondents are hereby
ordered to return immediately all documents, papers and other objects seized
or taken thereunder. Without costs.
6. Collecter vs Villaluz
7. Viduya vs. Berdiago, 73 SCRA 553

Post under case digests, Taxation at Monday, January 30, 2012 Posted by
Schizophrenic Mind
Facts: The search warrant issued by petitioner Viduya who was the former
Collector of Customs is quashed by the lower court upon motion by private
respondent Berdiago. The warrant of seizure and detention was issued on the
basis of reliable intelligence that fraudulent documents were used by
Berdiago in securing the release from the Bureau of Customs of a Rolls
Royce, it being made to appear that such car was a 1961 model instead of a
1966, thus enabling respondent to pay lower custom duties. There was a
demand for the correct amount due and Respondent expressed his
willingness to pay. Unfortunately, he was not able to live up to his promise so
a search warrant was issued, pursuant to Section 2099 of the Tariff and
Customs Code which requires a search warrant if such goods are located in a
dwelling house because the car was located in the Yabut Compound.
Moreover, it was not shown that Berdiago did not own the dwelling house
which was searched. Nonetheless, respondent judge quashed the warrant.
Issue: Whether or not there was grave abuse of discretion on the part of the
judge in quashing the search warrant? Yes
Held: Petition is granted. As the car was kept in a dwelling house in Wakas,
Barrio San Dionisio, Paraaque, Rizal, petitioner through two of his officers
in the Customs Police Service applied for and was able to obtain the search
warrant. Had there been no such move on the part of petitioner, the duties
expressly enjoined on him by law namely to assess and collect all lawful
revenues, to prevent and suppress smuggling and other frauds, and to
enforce tariff and customs law would not have been performed.

8. Dizon v Castro
The facts before the Court in these Certiorari, Prohibition, and mandamus
proceedings will be briefly stated. The three petitioners will be referred to
through their surnames of NOLASCO, AGUILAR-ROQUE and TOLENTINO.
1. Prior to August 6, 1984 (hereinafter to be referred to without the year),
AGUILAR-ROQUE was one of the accused of Rebellion in Criminal Case No.
MC-25-113 of Military Commission No. 25, both cases being entitled "People
of the Philippines vs. Jose Ma. Sison, et al." She was then still at large.
2. At 11:30 A.M. on August 6th, AGUILAR-ROQUE and NOLASCO were
arrested by a Constabulary Security Group (CSG) at the intersection of
Mayon Street and P. Margall Street, Quezon City. The stated time is an
allegation of petitioners, not denied by respondents. The record does not
disclose that a warrant of arrest had previously beeen issued against
NOLASCO.

3. At 12:00 N. on August 6th, elements of the CSG searched the premises at


239-B Mayon Street, Quezon City. The stated time is an allegation of
petitioners, not specifically denied by respondents. In their COMMENT,
however, respondents have alleged that the search was conducted "late on
the same day"; that is late on august 6th.
4. On August 6th, at around 9:00 A.M., Lt. Col. Virgilio G. Saldajeno of the
CSG, applied for a Search Warrant from respondent Hon. Ernani Cruz Pao,
Executive Judge of the Regional Trial Court in Quezon City, to be served at
No. 239-B Mayon Street, Quezon City, determined tyo be the leased residence
of AGUILAR-ROQUE, after almost a month of "round the clock surveillance"
of the premises as a "suspected underground house of the CPP/NPA."
AGUILAR-ROQUE has been long wanted by the military for being a high
ranking officer of the Communist Party of the Philippines, particularly
connected with the MV Karagatan/Doa Andrea cases.
In connection with the Search Warrant issued, the following may be stated:
(a) The Search Warrant was issued in proceedings entitled "People of the
Philippines vs. Mila Aguilar-Roque, Accused, Search Warrant No. 80- 84
for rebellion" (the SEARCH WARRANT CASE). Judge Panos Court was Branch
88.
(b) It does not appear from the records before us that an application in
writing was submitted by Lt. Col. Saldajeno to Judge Pao.
(c) According to the record, Lt. Col. Saldajeno and his witness S/A Dionicio A.
Lapus, were examined under oath by Judge Pao but only the deposition of
S/A Lapus has been submitted to us. The latter deposed that to his personal
knowledge, there were kept in the premises to be searched records,
documents and other papers of the CPP/NPA and the National Democratic
Front, including support money from foreign and local sources intended to be
used for rebellion. 1
5. In connection with the search made at 12:00 N. of August 6th the
following may be stated:
(a) TOLENTINO was a person then in charge of the premises. He was arrested
by the searching party presumably without a warrant of arrest.
(b) The searching party seized 428 documents and written materials, 2 and
additionally a portable typewriter, and 2 wooden boxes, making 431 items in
all. 3
(c) According to the Return, submitted in the SEARCH WARRANT CASE on
August 10th, 4 the search was made in the presence of Dra. Marciana
Galang, owner of the premises, and of two (2) Barangay Tanods. No mention
was made that TOLENTINO was present. The list of the 428 articles and
documents attached to the Return was signed by the two Barangay Tanods,
but not by Dra. Galang.
6. (a) On August 10th, the three petitioners, AGUILAR-ROQUE, NOLASCO
and TOLENTINO, were charged before the Quezon City Fiscal's Office (the
CITY FISCAL, for short) upon complaint filed by the CSG against petitioners

for "Subversion/Rebellion and/or Conspiracy to Commit


Rebellion/Subversion."
(b) On August 13th, the CITY FISCAL filed an Information for Violation of
Presidential Decree No. 33 (Illegal Possession of Subversive Documents)
against petitioners before Branch 42 of the Metropolitan Trial Court of
Quezon City (the SUBVERSIVE DOCUMENTS CASE), respondent Judge
Antonio P. Santos, presiding.
(c) On August 16th, CSG filed a Motion for Reconsideration with the CITY
FISCAL, praying that AGUILAR-ROQUE and NOLASCO be charged with
Subversion. The Motion was denied on November 16th.
7. (a) On September 10th, the CSG submitted an Amended Return in the
SEARCH WARRANT CASE praying, inter alia, that the CSG be allowed to
retain the seized 431 documents and articles, in connection with cases that
are presently pending against Mila Aguilar Roque before the Quezon City
Fiscal's Office and the court. 5
(b) On September 28th, petitioners were required by Judge Pano to comment
on the Amended Return, which AGUILAR-ROQUE did on October 18th,
raising the issue of the inadmissibility of any evidence obtained pursuant to
the Search Warrant.
(c) On December 13, 1984, Judge Pao admitted the Amended Return and
ruled that the seized documents "shall be subject to disposition of the
tribunal trying the case against respondent."
8. (a) On December 12th, petitioners filed a Motion to Suppress in the
SUBVERSIVE DOCUMENTS CASE, praying that such of the 431 items
belonging to them be returned to them. It was claimed that the proceedings
under the Search Warrant were unlawful. Judge Santos denied the Motion on
January 7, 1985 on the ground that the validity of the Search Warrant has to
be litigated in the SEARCH WARRANT CASE. He was apparently not aware of
the Order of Judge Pao of December 13th issued in the SEARCH WARRANT
CASE.
Hence, this Petition for Certiorari, Prohibition and mandamus to annul and
set aside the (1) Search Warrant issued by respondent RTC Judge Pao; (2)
his Order admitting the Amended Return and granting the Motion to Retain
Seized Items; and (3) Order of respondent MTC Judge Santos denying
petitioners' Motion to Suppress.
This Court, on February 12, 1985, issued a Temporary Restraining Order
enjoining the respondents or their duly authorized representatives from
introducing evidence obtained under the Search Warrant.
The PETITIONERS principally assert that the Search Warrant is void because
it is a general warrant since it does not sufficiently describe with
particularity the things subject of the search and seizure, and that probable
cause has not been properly established for lack of searching questions
propounded to the applicant's witness. The respondents, represented by the
Solicitor General, contend otherwise, adding that the questions raised cannot

be entertained in this present petition without petitioners first moving for the
quashal of the disputed Search Warrant with the issuing Judge.
We find merit in the Petition.
Section 3, Article IV of the Constitution, guarantees the right of the people to
be secure in their persons, houses, papers and effects against unreasonable
searches and seizures of whatever nature and for any purpose. It also
specifically provides that no Search Warrant shall issue except upon probable
cause to be determined by the Judge or such other responsible officer as may
be authorized by law, after examination under oath or affirmation of the
complainant and the witnesses he may produce, and particularly describing
the place to be searched and the things to be seized.
The disputed Search Warrant (No. 80-84) describes the personalities to be
seized as follows:
Documents, papers and other records of the Communist Party of the
Phihppines/New Peoples Army and/or the National Democratic Front, such
as Minutes of the Party Meetings, Plans of these groups, Programs, List of
possible supporters, subversive books and instructions, manuals not
otherwise available to the public, and support money from foreign or local
sources.
It is at once evident that the foregoing Search Warrant authorizes the seizure
of personal properties vaguely described and not particularized. It is an allembracing description which includes everything conceivable regarding the
Communist Party of the Philippines and the National Democratic Front. It
does not specify what the subversive books and instructions are; what the
manuals not otherwise available to the public contain to make them
subversive or to enable them to be used for the crime of rebellion. There is
absent a definite guideline to the searching team as to what items might be
lawfully seized thus giving the officers of the law discretion regarding what
articles they should seize as, in fact, taken also were a portable typewriter
and 2 wooden boxes. It is thus in the nature of a general warrant and
infringes on the constitutional mandate requiring particular description of
the things to be seized. In the recent rulings of this Court, search warrants of
similar description were considered null and void for being too general. Thus:
Subversive documents, pamphlets, leaflets, books, and other publications to
promote the objectives and purposes of the subversive organizations known
as Movement for Free Philippines. Light-a-Fire Movement and April 6
Movement. 6
The things to be seized under the warrant issued by respondent judge were
described as 'subversive documents, propaganda materials, FAs, printing
paraphernalia and all other subversive materials Such description hardly
provided a definite guideline to the search team as to what articles might be
lawfully seized thereunder. Said description is no different from if not worse
than, the description found in the search warrants in "Burgos, et al. v. the
Chief of Staff"which this Court declared null and void for being too general. 7

In the case at bar, the search warrant issued by respondent judge allowed
the seizure of printed copies of the Philippine Times, manuscripts/drafts of
articles for publication, newspaper dummies subversive documents, articles,
etc., and even typewriters, duplicating machines, mimeographing and tape
recording machines. Thus, the language used is so all embracing as to
include all conceivable records and equipment of petitioner regardless of
whether they are legal or illegal. The search warrant under consideration was
in the nature of a general warrant which is constitutionally objectionable. 8
The lack of particularization is also evident in the examination of the witness
presented by the applicant for Search Warrant.
Q Mr. Dionicio Lapus, there is an application for search warrant filed by Lt.
Col. Virgilio Saldajeno and the Court would like to know if you affirm the
truth of your answer in this deposition?
(The deposition instead)
A Yes, sir,
Q How long did it take you for the surveillance?
A Almost a month, sir.
Q Are you a lawyer, Mr. Lapus?
A No, Your Honor, but I was a student of law.
Q So, you are more or less familiar with the requisites of the application for
search warrant?
A Yes, Your Honor.
Q How did you come to know of the person of Mila Aguilar-Roque?
A Because of our day and night surveillance, Your Honor, there were so many
suspicious persons with documents.
Q What kind of documents do you refer to?
A Documents related to the Communist Party of Philippines and New People's
Army.
Q What else?
A Conferences of the top ranking officials from the National Democratic
Front, Organization of the Communist Party of the Philippines ...
Q And may include what else?
A Other papers and documents like Minutes of the Party Meetings, Plans of
these groups, Programs, List of possible supporters, subversive books and
instructions, manuals not otherwise available to the public and support
money from foreign and local sources. 9
The foregoing questions propounded by respondent Executive Judge to the
applicant's witness are not sufficiently searching to establish probable cause.
The "probable cause" required to justify the issuance of a search warrant
comprehends such facts and circumstances as will induce a cautious man to
rely upon them and act in pursuant thereof. 10 Of the 8 questions asked, the
1st, 2nd and 4th pertain to Identity. The 3rd and 5th are leading not
searching questions. The 6th, 7th and 8th refer to the description of the
personalities to be seized, which is Identical to that in the Search Warrant

and suffers from the same lack of particularity. The examination conducted
was general in nature and merely repetitious of the deposition of said
witness. Mere generalization will not suffice and does not satisfy the
requirements of probable cause upon which a warrant may issue. 11
Respondents claim, however, that the proper forum for questioning the
illegality of a Search Warrant is with the Court that issued it instead of this
original, independent action to quash. The records show, however, that
petitioners did raise that issue in the SEARCH WARRANT CASE in their
Comment, dated October 18, 1984. In fact, they already questioned the
admissibility of the evidence obtained under the Search Warrant, even during
the inquest investigation on August 10, 1984. And in the SUBVERSIVE
DOCUMENTS CASE, they filed a Motion to Suppress on December 12, 1984
claiming that the proceedings under the Search Warrant were unlawful.
Substantially, therefore, while not denominated as a motion to quash,
petitioners had questioned the legality of the Search Warrant.
Parenthetically, it strikes the Court that the pendency of the SEARCH
WARRANT CASE and of the SUBVERSIVE DOCUMENTS CASE before two
different Courts is not conducive to an orderly administration of justice. It
should be advisable that, whenever a Search Warrant has been issued by one
Court, or Branch, and a criminal prosecution is initiated in another Court, or
Branch, as a result of the service of the Search Warrant, the SEARCH
WARRANT CASE should be consolidated with the criminal case for orderly
procedure. The later criminal case is more substantial than the Search
Warrant proceeding, and the Presiding Judge in the criminal case should
have the right to act on petitions to exclude evidence unlawfully obtained.
Notwithstanding the irregular issuance of the Search Warrant and although,
ordinarily, the articles seized under an invalid search warrant should be
returned, they cannot be ordered returned in the case at bar to AGUILARROQUE. Some searches may be made without a warrant. Thus, Section 12,
Rule 126, Rules of Court, explicitly provides:
Section 12. Search without warrant of person arrested.A person charged
with an offense may be searched for dangerous weapons or anything which
may be used as proof of the commission of the offense.
The provision is declaratory in the sense that it is confined to the search,
without a search warrant, of a person who had been arrested. It is also a
general rule that, as an incident of an arrest, the place or premises where the
arrest was made can also be search without a search warrant. In this latter
case, "the extent and reasonableness of the search must be decided on its
own facts and circumstances, and it has been stated that, in the application
of general rules, there is some confusion in the decisions as to what
constitutes the extent of the place or premises which may be
searched. 12 "What must be considered is the balancing of the individual's
right to privacy and the public's interest in the prevention of crime and the
apprehension of criminals." 13

Considering that AGUILAR-ROQUE has been charged with Rebellion, which


is a crime against public order; that the warrant for her arrest has not been
served for a considerable period of time; that she was arrested within the
general vicinity of her dwelling; and that the search of her dwelling was made
within a half hour of her arrest, we are of the opinion that in her respect, the
search at No. 239-B Mayon Street, Quezon City, did not need a search
warrant; this, for possible effective results in the interest of public order.
Such being the case, the personalities seized may be retained. by CSG, for
possible introduction as evidence in the Rebellion Case, leaving it to
AGUILAR-ROQUE to object to their relevance and to ask Special Military
Commission No.1 to return to her any and all irrelevant documents and
articles.
WHEREFORE, while Search Warrant No. 80-84 issued on August 6, 1984 by
respondent Executive Judge Ernani Cruz Pao is hereby annulled and set
aside, and the Temporary Restraining Order enjoining respondent from
introducing evidence obtained pursuant to the Search Warrant in the
Subversive Documents case hereby made permanent, the, personalities
seized may be retained by the Constabulary Security Group for possible
introduction as evidence in Criminal Case No. SMC-1-1, pending before
Special Military commission No. 1, without prejudice to petitioner Mila
Aguilar-Roque objecting to their relevance and asking said Commission to
return to her any and all irrelevant documents and articles.
9. People vs Veloso
-In May, 1923, the building located at No. 124 Calle Arzobispo, City of
Manila, was used by anorganization known as the Parliamentary Club. Jose
Ma. Veloso was at that time a member of theHouse of Representative of the
Philippine Legislature. He was also the manager of the club.-The police of
Manila had reliable information that the so-called Parliamentary Club was
nothingmore than a gambling house. Indeed, on May 19, 1923, J. F.
Townsend, the chief of the gamblingsquad, had been to the club and verified
this fact. As a result, on May 25, 1923, Detective AndresGeronimo of the
secret service of the City of Manila, applied for, and obtained a search
warrantfrom Judge Garduo of the municipal court. Thus provided, the
police attempted to raid theParliamentary Club a little after three in the
afternoon of the date above- mentioned. They foundthe doors to the premises
closed and barred. Accordingly, one band of police including
policemanRosacker, ascended a telephone pole, so as to enter a window of
the house. Other policemen,headed by Townsend, broke in the outer door.Once inside the Parliamentary Club, nearly fifty persons were apprehended
by the police. One of them was the defendant Veloso. Veloso asked Townsend
what he wanted, and the latter showedhim the search warrant. Veloso read it
and told Townsend that he was Representative Veloso andnot John Doe, and
that the police had no right to search the house. Townsend answered

thatVeloso was considered as John Doe. As Veloso's pocket was bulging, as if


it contained gamblingutensils, Townsend required Veloso to show him the
evidence of the game. About five minuteswas consumed in conversation
between the policemen and the accused the policemen insistingon searching
Veloso, and Veloso insisting in his refusal to submit to the search.-At last the
patience of the officers was exhausted. So policeman Rosacker took hold of
Velosoonly to meet with his resistance. Veloso bit Rosacker in the right
forearm, and gave him a blow inanother part of the body, which injured the
policeman quite severely. Through the combinedefforts of Townsend and
Rosacker, Veloso was finally laid down on the floor, and long sheets of paper,
of reglas de monte, cards, cardboards, and chips were taken from his
pockets.-All of the persons arrested were searched and then conducted to the
patrol wagons. Velosoagain refused to obey and shouted offensive epithets
against the police department. It wasnecessary for the policemen to conduct
him downstairs. At the door, Veloso resisted sotenaciously that three
policemen were needed to place him in the patrol wagon.
-The warrant read as follows:
SEARCH WARRANT (G)
The People of the Philippine Islands, to any member of the Police Force of the
City of Manila.
GREETINGS
Proof by affidavit having this day been made before me by Andres Geronimo
that hehas good reason to believe and does believe that John Doe has
illegally in hispossession in the building occupied by him and which is under
his control, namely inthe building numbered 124 Calle Arzobispo, City of
Manila, Philippines Islands, certaindevices and effects used in violation of
the Gambling Law, to wit: money, cards,chips, reglas, pintas, tables and
chairs and other utensils used in connection with thegame commonly known
as monte and that the said John Doe keeps and conceals saiddevices and
effects with the illegal and criminal intention of using them in violation of the
Gambling Law.Now therefore, you are hereby commanded that at any time in
the day or nightwithin ten (10) days on or after this date to make a search on
the person of said JohnDoe and in the house situated at No. 124 Calle
Arzobispo, City of Manila, PhilippineIslands, in quest of the above described
devices and effects and if you find the sameor any part thereof, you are
commanded to bring it forthwith before me as providedfor by law.Given under
my hand, this 25th day of May, 1923.
(Sgd.) L. GARDUO Judge, Municipal Court
Issue:
WON the search warrant and the arrest of Veloso was valid.
Ruling:
Yes.
RD:

It is provided, among other things, in the Philippine Code on Criminal


Procedure that a searchwarrant shall not issue except for probable cause
and upon application supported by oathparticularly describing the place to
be searched and the person of thing to be seized. The name and description
of the accused should be inserted in the body of the warrant andwhere the
name is unknown there must be such a description of the person accused as
willenable the officer to identify him when found.A warrant for the
apprehension of a person whose true name is unknown, by the name of
"JohnDoe" or "Richard Roe," "whose other or true name in unknown," is void,
without other and furtherdescriptions of the person to be apprehended, and
such warrant will not justify the officer inacting under it. Such a warrant
must, in addition, contain the best descriptio personae possibleto be
obtained of the person or persons to be apprehended, and this description
must be sufficient to indicate clearly the proper person or persons upon
whom the warrant is to beserved; and should state his personal appearance
and peculiarities, give his occupation andplace of residence, and any other
circumstances by means of which he can be identified.In the first place, the
affidavit for the search warrant and the search warrant itself described
thebuilding to be searched as "the building No. 124 Calle Arzobispo, City of
Manila, PhilippineIslands." This, without doubt, was a sufficient designation
of the premises to be searched.As the search warrant stated that John Doe
had gambling apparatus in his possession in thebuilding occupied by him at
No. 124 Calle Arzobispo, City of Manila, and as this John Doe was Jose Ma.
Veloso, the manager of the club, the police could identify John Doe as Jose
Ma. Velosowithout difficulty.
ELI LUI, ET AL. VS. MATILLANO, May 27, 2004
Right against unreasonable searches and seizures; Mission Order does
not authorize an illegal search. Waiver of the right against an
unreasonable search and seizure.
In search of the allegedly missing amount of P45,000.00 owned by the
employer, the residence of a relative of the suspect was forcibly open by the
authorities by kicking the kitchen door to gain entry into the house.
Thereafter, they confiscated different personal properties therein which were
allegedly part of those stolen from the employer. They were in possession of a
mission order but later on claimed that the owner of the house gave his
consent to the warrantless search. Are the things admissible in evidence?
Can they be sued for damages as a result of the said warrantless search and
seizure?
Held:
The right against unreasonable searches and seizures is a personal right
which may be waived expressly or impliedly. BUT A WAIVER BY
IMPLICATION CANNOT BE PRESUMED. There must be clear and convincing

evidence of an actual intention to relinquish the right. There must be proof of


the following:
a.
that the right exists;
b.
that the person involved had knowledge, either constructive or
actual, of the existence of said right;
c.
that the said person had an actual intention to relinquish the right.

Finally, the waiver must be made voluntarily, knowingly and intelligently in


order that the said is to be valid.
The search was therefore held illegal and the members of the searching party
held liable for damages in accordance with the doctrine laid down in Lim vs.
Ponce de Leon and MHP Garments vs. CA.a

You might also like